LSAT and Law School Admissions Forum

Get expert LSAT preparation and law school admissions advice from PowerScore Test Preparation.

 afinelli
  • Posts: 26
  • Joined: Sep 05, 2011
|
#1950
I am completely stumped with this question! I went with answer choice E. How is B correct?
Thanks for any help!
 Steve Stein
PowerScore Staff
  • PowerScore Staff
  • Posts: 1153
  • Joined: Apr 11, 2011
|
#1966
That's a tough one. An unwealthy candidate who wants to win non-fully-subsidized democratic elections needs wealthy patrons—so it’s plausible that the candidate might compromise on some views to keep these wealthy supporters happy.

The author doesn’t think this is a problem, though, because the wealthy are distributed among the population and among the political parties in roughly the same way, so each of the parties has its fair share of wealthy patrons. If a democrat wants wealthy supporters, for example, he doesn’t have to change his position to get support because the party has its fair share of wealthy patrons.

The problem with this argument is that a candidate might represent a given party and still conceivably compromise views regarding some specific issues. This is what correct answer choice B provides: the author doesn’t consider the fact that a candidate might take positions on many more varied issues than the party—so a candidate could be aligned with the party on all of its chosen political positions but could still potentially compromise on other issues to win over wealthy supporters.
 afinelli
  • Posts: 26
  • Joined: Sep 05, 2011
|
#1973
Ok - got it. I wasn't thinking about the parties vs. more varied positions. Thank you so much for the explanation.
 Nina
  • Posts: 81
  • Joined: Sep 11, 2012
|
#5703
I got lost in this question. Does the stimulus argue that, since the wealthy are of equal proportion among various parties, the situation that "candidates will compromise their views" is not likely to occur? and why choose answer B will cause the argument vulnerable?

Thanks a lot!
 Steve Stein
PowerScore Staff
  • PowerScore Staff
  • Posts: 1153
  • Joined: Apr 11, 2011
|
#5715
Hi Nina,

In that one, the author says that there should not be an issue of poor candidates' selling out their positions to wealthy patrons, because the wealthy are dispersed in equal proportions among the political parties. But if there are many more positions than political parties, the referenced proportionality is not much consolation--a democratic candidate, for example, might still have to make serious compromise on several positions in order to placate even a democratic patron--even a supporter from the same party.

This is a very difficult question--let me know whether this clears it up--thanks!

~Steve
 Nina
  • Posts: 81
  • Joined: Sep 11, 2012
|
#5744
yeah, right now it makes much more sense to me. Thanks, Steve!
 Applesaid
  • Posts: 29
  • Joined: Oct 18, 2013
|
#12185
Not sure what the correct answer means exactly tho I am pretty sure it is the correct answer choice!

I reduce the argument as below:

premise: the wealthy are dispersed among the various political parties in roughly equal proportion to their percentage in the overall population.

conclusion: non-wealthy candidates supported by the wealthy will not compromise their views to win the support.

Ok, so

1) I am not sure why the premise was put forward like that. It doesn't seem to justify the conclusion.

2) not sure what exactly the correct answer choice means by saying that positions endorsed by political parties might be much less varied than the positions taken by candidates (what kind of candidates? wealthy? non wealthy?)
 Jacques Lamothe
PowerScore Staff
  • PowerScore Staff
  • Posts: 50
  • Joined: Sep 24, 2013
|
#12216
Hey Applesaid,

You are right to pick up on the disconnect from the stated premise and the conclusion. The author's argument definitely relies on an assumption that since wealthy persons are proportionally distributed across political parties, candidates will receive financial support regardless of their political views. That leads to the author's conclusion that candidates will not need to change their views to get financial support.

Once you identify the assumption, it is easier to see how answer choice (B) is a good criticism of the argument. If the views of candidates vary much more than political parties that wealthy patrons belong to, it is likely that some candidates will have views that do not align with any wealthy patrons. To take an extreme examples, imagine a proposed government regulation that both parties of a 2-party system support. If candidates have views that vary more than the parties, there will be some candidates who do not support the proposed regulation. Then, even though wealthy patrons are distributed across parties, they are not evenly distributed across all possible candidate positions. Some candidates (in this example, ones that do not support the policy) may feel pressure to change their views since none of the wealthy patrons share their current views, which contradicts the conclusion in the stimulus.

I hope that helps!

Jacques
 Applesaid
  • Posts: 29
  • Joined: Oct 18, 2013
|
#12221
Thanks for your reply! This is, perhaps, the most twisted way of thinking I've ever had.
 karen_k
  • Posts: 35
  • Joined: Sep 24, 2015
|
#22321
Hi,

I am quite lost on both of these questions. On 22, I don't understand how B is correct. I'd greatly appreciate any clarification! Thank you!

Get the most out of your LSAT Prep Plus subscription.

Analyze and track your performance with our Testing and Analytics Package.